Marking Scheme

You might also like

Download as pdf or txt
Download as pdf or txt
You are on page 1of 22

2017 Mock Paper (Compulsory Part) - Paper 1 (Marking Scheme)

Solution Marks Remarks

(m 1n 2 ) 3
1.
m 4 n 5

m 3 n 6 for (ab)m  ambm or (am)n  amn


= 1M
m 4 n 5
cp pq cp 1
= m 3( 4) n 65 1M for q  c or q  q  p
c c c
= mn 1A
(3)
2. (a) a  2ab  3b
2 2

 (a  b)( a  3b) 1A or equivalent

(b) a 2  2ab  3b 2  a  b
 a 2  2ab  3b 2  (a  b)
 (a  b)( a  3b)  (a  b) 1M for using the result of (a)
 (a  b)( a  3b  1) 1A or equivalent

(3)
2 3
3. x 
y z
3 2
x  1M for putting y on one side
z y
xz  3 2
 1M
z y
z y

xz  3 2
2z
y 1A or equivalent
xz  3
(3)

4. (a) x  3( x  4)  10
x  3x  12  10 1M
 2 x  2
x 1 1A

Therefore, we have x  3 or x  1 .
Thus, the solutions of (*) are all real numbers. 1A
(b) 1 1A
(4)

2017-DSE-MATH-CP 1-MS 1 © Pearson Education Asia Limited 2016


2017 Mock Paper (Compulsory Part) - Paper 1 (Marking Scheme)

Solution Marks Remarks


5. Let $x and $y be the the prices of an apple and an orange
respectively.
 x  3 y  20
 1A
 y  2 x  18
So, we have x  3(18  2 x)  20 . 1M for putting a linear equation in x
or y only
Solving, we have x  6.8 and y  4.4 . 1A
Thus, the required total price
 $[ 2(6.8)  3(4.4)]
 $26 .8 1A
(4)

6. (a) 25 000  (1  r%  3)  29 875 1M


1  0.03r  1.195
0.03r  0.195
r  6.5 1A

Alternative Solution
Interest  $(29 875  25 000 )
 $4875
25 000  r%  3  4875 1M
r%  0.065
r  6.5 1A

(b) Amount she will get back from bank B after 3 years
312
 6% 
 $25 000  1   1M
 12 
 $29917
∵ Amount she will get back from bank B > amount
she will get back from bank A
i.e. Interest she will receive from bank B > interest
she will received from bank A
∴ She should choose bank B. 1A f.t.
(4)

2017-DSE-MATH-CP 1-MS 2 © Pearson Education Asia Limited 2016


2017 Mock Paper (Compulsory Part) - Paper 1 (Marking Scheme)

Solution Marks Remarks

7. (a) Let be the angle of the sector.



   10 2  2 1M
360 
  7.2
Thus, the angle of the sector is 7.2°. 1A

(b) Perimeter of the sector


 7.2 
  2  10  10  2  cm 1M
 360  
 (0.4  20) cm 2 
1A    20  cm
 5 

(4)
8. (a) AOB  228   138 
 90  1A
OA  3 and OB  1
AB 2  OA 2  OB 2 (Pyth. theorem)
AB  OA  OB 2 2

 ( 3 ) 2  12
2 1A
OB
sin OAB 
AB
1
sin OAB 
2
OAB  30  1A

(b) (2, 108°) and (2, 168°) 1A + 1A 1A for one pair + 1A for all
(5)
9. (a) Join BC. E
D
Note that EBC  90 . (∠ in semi-circle) 1A
A
Also, note that
ABC  ADC  180  . (opp. ∠s, cyclic quad.)
30°
So, we have ADC  180   30  90
B C
Therefore, we have ADC  60 . 1A

2017-DSE-MATH-CP 1-MS 3 © Pearson Education Asia Limited 2016


2017 Mock Paper (Compulsory Part) - Paper 1 (Marking Scheme)

Solution Marks Remarks

Alternative Solution
Join DE. E
D
Note that
A
ADE  ABE  30 . (∠s in the same segment)
Also, note that CDE  90 . (∠ in semi-circle) 1A
30°
ADC  90   30 
 60  B C
1A

(b) Note that DCA  DAC . (base ∠s, isos. △)


Also, note that
DCA  DAC  ADC  180. (∠ sum of △)
180   60
So, we have DCA   60  . 1A
2
Note that
ECA  EBA  30 . (∠s in the same segment) 1A
So, we have ECD  60  30  30 .
∵ ECA  ECD
∴ CE is the angle bisector of ∠ACD.
Thus, the claim is correct. 1A f.t.
(5)
10. (a) DCE  DCB (given)
EDC  DCB (alt. ∠s, DE // BC)
∴ DCE  EDC
∴ DE  CE (sides opp. equal ∠s)

Marking Scheme:
Case 1 Any correct proof with correct reasons. 2
Case 2 Any correct proof without reasons. 1

(b) ∵ △ADE ~ △ABC (AAA)


∴ AE : CE  AD : BD (corr. sides, ~△s)
 5:3 1A

DE
sin BAC  1M
AE
CE
sin BAC 
AE
3
sin BAC 
5
BAC  36 .9 1A

(5)

2017-DSE-MATH-CP 1-MS 4 © Pearson Education Asia Limited 2016


2017 Mock Paper (Compulsory Part) - Paper 1 (Marking Scheme)

Solution Marks Remarks


11. (a) Let $C be the painting cost of a solid and S cm2 be its
surface area.
Let C  k1  k 2 S , where k1 and k2 are non-zero
constants. 1A
So, we have 300  k1  k 2 (200 ) and
360  k1  k 2 (250 ) . 1A for either substitution
Solving, we have k1  60 and k 2  1.2 . 1A for both correct
Painting cost of solid A
 $[ 60  1.2(605 )]
 $786 1A

(b) Let VA cm3 and VB cm3 be the volumes of solids A and B


respectively, and let SA cm2 and SB cm2 be the surface
area of solids A and B respectively.
V A  (1  33 .1%) VB
 1.331 VB

Note that solids A and B are similar.


3
VA SA
 1M
VB SB
3
S 
1.331 2   A 
 SB 
SA
 1.21
SB
S A  1.21S B

605  1.21S B
S B  500 1A
Painting cost of solid B
 $[ 60  1.2(500 )]
 $660
786  660
100 %  19 .1% 1A
660
 20 %
Thus, the claim is incorrect. 1A f.t.
(8)

2017-DSE-MATH-CP 1-MS 5 © Pearson Education Asia Limited 2016


2017 Mock Paper (Compulsory Part) - Paper 1 (Marking Scheme)

Solution Marks Remarks


12. (a) (i) Slope of L1
4 .5  0

0  ( 9 )
1
 1A
2
Slope of L2
1 either one

( 2)
1

2
∵ Slope of L1 = slope of L2
∴ L1 and L2 are parallel. 1A f.t.
(ii) y-intercept of 
 3
4. 5    
 2
 1M
2
3

2
Note that  is parallel to L1 and L2. 1M
1
Slope of  
2
The equation of  is
1 3
y x 1A or equivalent
2 2
(b) Note that the centre of C is (0, n). 1A
Since AB  HK , the perpendicular distances from
centre to AB and HK are equal. 1M
So, the centre lies on .

1 3
n (0) 
2 2
3
 1A
2

(8)
13. (a) f ( x)  ( x  1)(x  h)  4x  k , where h and k are
2
1A
non-zero constants.
∵ f (x) is divisible by x + 1.
∴ f (1)  0 1M

∴ [( 1)  1]( x  h)  4(1)  k  0


2

k 4 1A

2017-DSE-MATH-CP 1-MS 6 © Pearson Education Asia Limited 2016


2017 Mock Paper (Compulsory Part) - Paper 1 (Marking Scheme)

Solution Marks Remarks


(b) ∵ f ( x)  0
∴ ( x 2  1)( x  h)  4 x  4  0
( x  1)( x  1)( x  h)  4( x  1)  0
( x  1)[( x  1)( x  h)  4]  0
x 1  0 or x 2  (h  1) x  4  h  0
x  1 or x 2  (h  1) x  4  h  0 1A
Since there are two distinct real roots, there are two
cases.
Case 1: x 2  (h 1) x  4  h  0 has two distinct roots
and one of them is x  1 .
(1) 2  (h  1)( 1)  4  h  0 1M
h3
Since   (3  1) 2  4(4  3)  0 ,
x 2  (h 1) x  4  h  0 has repeated roots –1
when h  3 .
Thus, h  3 and there are no value of h satisfying
case 1.
Case 2: x 2  (h 1) x  4  h  0 has repeated roots
which are not equal to –1.

(h  1) 2  4(4  h)  0 1M
h  2h  1  16  4h  0
2

h 2  2h  15  0
(h  5)( h  3)  0
h  5 or h  3 (rejected)

Thus, the possible value of h is –5. 1A


(7)
14. (a) a 5, b 0 1A + 1A

(b) The mean  61 1A


The standard deviation  6.3875  6.39 1A
(c) (i) Note that two newly added scores are
symmetrical about the mean.
So, we have the mean of the scores of all
22 customers remains unchanged.
The mean  61 1A

Alternative Solution

61  20  (61  p )  (61  p )
The mean 
22
 61 1A

2017-DSE-MATH-CP 1-MS 7 © Pearson Education Asia Limited 2016


2017 Mock Paper (Compulsory Part) - Paper 1 (Marking Scheme)

Solution Marks Remarks


(ii) The standard deviation of the scores of all 22
customers
6.3875 2  20  (61  p  61) 2  (61  p  61) 2

22
6.3875 2  20  2 p 2

22
6.3875 2  20  2(6.3875 2 )
 1M
22
 6.3875
So, the standard deviation is increased.
Thus, the claim is correct. 1A f.t.
(7)
15. (a) Note that the mean of the standard scores of the
students in the class is zero.
Since after excluding the four students, the mean of the
scores does not change, the mean of the standard scores
of the four students is zero.

3.1  0.1  k  0.1  k  k


0 1M
4
k   3.3 1A

Alternative Solution
Let a, b, c and d be the scores of the four students, N,
x and  be the number of students, the mean and the
standard deviation of the scores of the students
respectively.

N  x  (a  b  c  d )
x
N 4
a  b  c  d  4x

Sum of the standard scores of the four students


(a  b  c  d )  4 x


4x  4x


0
∴ 3.1  0.1  k  0.1  k  k  0 1M
k   3.3 1A

2017-DSE-MATH-CP 1-MS 8 © Pearson Education Asia Limited 2016


2017 Mock Paper (Compulsory Part) - Paper 1 (Marking Scheme)

Solution Marks Remarks


(b) Let  be the standard deviation of the scores of
the students.
74  40
0.1  (3.3) 

34

3.4
 10 1A
Standard score of Peter
72  40

10
 3.2
 3.1
Thus, the claim is incorrect. 1A f.t.
(4)
16. (a) The required probability

C34C16  C44
 1M for numerator
C410
5
 1A r.t. 0.119
42

Alternative Solution
The required probability

4 3 2 1 4 3 2 6 for p1 p2 p3 p4  4 p1 p2 p3 p5
     4    1M
10 9 8 7 10 9 8 7
5
 1A r.t. 0.119
42

Alternative Solution
The required probability

C46 C14C36 C24C26


 1  10  10 1M for 1  p6  p7  p8
C410 C4 C4
5
 1A r.t. 0.119
42

(b) (i) The required probability

C22C12C12
 1M for numerator
C410
2
 1A r.t. 0.0190
105

2017-DSE-MATH-CP 1-MS 9 © Pearson Education Asia Limited 2016


2017 Mock Paper (Compulsory Part) - Paper 1 (Marking Scheme)

Solution Marks Remarks

Alternative Solution
The required probability

2 1 1 2
 24     1M for 24 p9 p10 p11 p12
10 9 8 7
2
 1A r.t. 0.0190
105

(ii) The required probability


2
 105 1M
5
42
4
 1A
25

(6)
60
17. (a) Slope of L1 
60
1
The equation of L1 is y  x . 1A
x  y  0

Thus, the system of inequalities is 2 x  3 y  30 . 1M + 1A or equivalent
x  0

(b) Let x and y be the numbers of apples and oranges Mary


buys respectively.
Now, the constraints are 2x  3y  30 and x  y ,
where x and y are non-negative integers.
Denote the total number of fruit by N.
Then, we have N  x  y . 1A
Note that the vertices of the shaded region in Figure 3
are points (0, 0), (0, 10) and (6, 6).
At the point (0, 0), we have N  0  0  0 . 1M for testing all points
At the point (0, 10), we have N  0  10  10 .
At the point (6, 6), we have N  6  6  12 .
So, the greatest possible total number of fruit is 12.
Therefore, the total number of fruit bought cannot
exceed 12.
Thus, the claim is incorrect. 1A f.t.
(6)
2017-DSE-MATH-CP 1-MS 10 © Pearson Education Asia Limited 2016
2017 Mock Paper (Compulsory Part) - Paper 1 (Marking Scheme)

Solution Marks Remarks

18. (a) An1  An  log 2 (a  2 n11 )  log 2 (a  2 n1 )


 log 2 a  log 2 2 n  (log 2 a  log 2 2 n1 )
 log 2 a  n log 2 2  log 2 a  (n  1) log 2 2
 log 2 2
 1 , which is a constant

So, A1, A2, A3, … + An is an arithmetic sequence. 1M


11
A1  log 2 (a  2 )  log 2 a
10
[2 log 2 a  (10  1)(1)]  75 1M
2
2 log 2 a  9  15
2 log 2 a  6
log 2 a  3
a  23
8 1A

(b) A1  3
A1  A2  A3  ...  An  100
n
[2(3)  (n  1)(1)]  100
2
n(n  5)  200
n 2  5n  200  0
  
 n   5  5 33  n   5  5 33   0 1M
 2  2 
  
 5  5 33  5  5 33
n or n  1M
2 2
Note that n is a positive integer.
Thus, the smallest value of n is 12. 1A
(6)

2017-DSE-MATH-CP 1-MS 11 © Pearson Education Asia Limited 2016


2017 Mock Paper (Compulsory Part) - Paper 1 (Marking Scheme)

Solution Marks Remarks


19. (a) Note that G is the circumcentre of △ABC and
AE  CE .
So, we have GE  AC .
Let J be the mid-point of AB. Similarly, GJ  AB .
In △AEG and △AJG,
∵ AEG  AJG  90
AG  AG (common side)

8
AE  AJ  cm  4 cm
2
∴ △AEG  △AJG (RHS)
∴ EAG  JAG (corr. ∠s, △s) 1M
Also, AB = AC.
So, we have AD  BC and
BD  CD . (prop. of isos. △) 1M
∵ △AEG ~ △ADC (AAA)
AE AG
∴  (corr. sides, ~△s)
AD AC
4 cm 5 cm

AD 8 cm
AD  6.4 cm 1A
In △ADC,
AD 2  CD 2  AC 2 (Pyth. theorem)
CD  AC 2  AD 2
 8 2  6.4 2 cm
 4.8 cm 1A

(b) (i) By the Pythagoras’ theorem, we have


D G
EG  AE  AG ,
2 2 2

A
EH 2  EG 2  GH 2 , 1M
B H E
AH 2  AG 2  GH 2 .
C
So, we have

EH 2  AE 2  EG 2  GH 2  AE 2
 EG 2  AE 2  GH 2
 AG 2  GH 2
 AH 2
Thus, we have AEH  90 . 1A f.t.
(converse of Pyth. theorem)

2017-DSE-MATH-CP 1-MS 12 © Pearson Education Asia Limited 2016


2017 Mock Paper (Compulsory Part) - Paper 1 (Marking Scheme)

Solution Marks Remarks


(ii) Note that H is the incentre of △ABC.
D G
So, we have FAB  FAC .
A
Also, note that AB  AC .
B H
F E
So, we have AF  BC . (prop. of isos. △)
C
Since HF  BC , HE  AC and H is the
incentre of △ABC, HE  HF .
In △CEH and △CFH,
∵ CFH  CEH  90 (proved)
CH  CH (common side)
HE  HF (proved)
Thus, we have △CEH  △CFH. (RHS)

Marking Scheme:
Case 1 Any correct proof with correct reasons. 2
Case 2 Any correct proof without reasons. 1

(iii) Note that △CEH  △CFH.


So, we have CF  CE  4 cm .
Note that BF  CF . (prop. of isos. △)
So, we have BC  2  4 cm  8 cm .
In △BCD,

4.8 2  4.8 2  8 2
cos BDC  1M
2(4.8)( 4.8)
BDC  112 .8853805 

Area of △BDC
1
 (4.8)( 4.8) sin 112 .8853805  cm 2
2
 10 .61319933 cm 2
Since ADC  90 and ADB  90 , AD is
perpendicular to the plane BCD. 1M
Volume of the pyramid ABCD
1
  area of △BDC  AD
3
1
  10 .61319933  6.4 cm3
3
 22 .6 cm3 1A

2017-DSE-MATH-CP 1-MS 13 © Pearson Education Asia Limited 2016


2017 Mock Paper (Compulsory Part) - Paper 1 (Marking Scheme)

Solution Marks Remarks

Alternative Solution
Note that △CEH  △CFH.
So, we have CF  CE  4 cm .
Note that BF  CF . (prop. of isos. △)
So, we have BC  2  4 cm  8 cm .

BC  CD  BD
Let s  .
2

8  4.8  4.8
∴ s  8.8
2
Area of △BDC
 8.8(8.8  8)(8.8  4.8)(8.8  4.8) cm 2 1M
 10 .61319933 cm 2
Since ADC  90 and ADB  90 , AD is
perpendicular to the plane BCD. 1M
Volume of the pyramid ABCD
1
  area of △BDC  AD
3
1
  10 .61319933  6.4 cm3
3
 22 .6 cm3 1A

(c) Let BDC   .


Volume of the pyramid ABCD
1 1 
    4.8  4.8  sin    6.4 cm 3 1M
3 2 
Since sin  is the greatest when θ = 90°,
the volume of the pyramid ABCD is the greatest when
θ = 90°.
Thus, the claim is correct. 1A f.t.
(13)

2017-DSE-MATH-CP 1-MS 14 © Pearson Education Asia Limited 2016


2017 Mock Paper (Compulsory Part) - Paper 2 (Answers)

Paper 2
Answers

Question No. Key Question No. Key


Section A 1. C Section B 31. A
2. D 32. B
3. C 33. A
4. D 34. D
5. C 35. C

6. A 36. B
7. B 37. A
8. D 38. B
9. C 39. C
10. B 40. C

11. C 41. A
12. D 42. C
13. A 43. B
14. B 44. C
15. D 45. D

16. D
17. D
18. A
19. C
20. C

21. D
22. A
23. B
24. D
25. D

26. A
27. B
28. A
29. B
30. A

© Pearson Education Asia Limited –1–


Longman Mathematics Series 2017 Mock Paper (Compulsory Part) - Paper 2
2017 Mock Paper (Compulsory Part) - Paper 2 (Full Solutions)

Paper 2
Full Solutions
6. A
Section A y  ax 2  x  b
1. C
∵ The graph opens downwards.
( x  1)( x 2  2 x  1)  x 3  2 x 2  x  x 2  2 x  1 ∴ a < 0
 x3  x 2  x 1 i.e. a > 0
∵ y-intercept of the graph = b
2. D ∴ b < 0
(9k 2 ) 2 81k 4 i.e. b > 0
 ∴ The answer is A.
3k 2 3k 2
27 7. B
 6
k For I:
∵ ab
3. C 1 1
∴ 
2h  k  5  (1) a b
 ∴ I is not true.
4h  3k  5  (2) For II:
From (1), we have ∵ b > 0 and k < 0
k  2h  5 (3)
1 1
By substituting (3) into (2), we have ∴  0 and  0
b k
4h  3(2h  5)  5
1 1
10h  20 i.e. 
b k
h  2 ∴ II must be true.
By substituting h = 2 into (3), we have For III:
k  2(2)  5 ∵ a > b and k < 0
1 ∴ ak < bk
∴ III is not true.
4. D ∴ The answer is B.
L.H.S.  ( 4  mx)( x  1)  nx
8. D
 4 x  4  mx 2  mx  nx
2x
  mx 2  (4  m  n) x  4  5  3x  1 1  3
and
3
R.H.S.  nx(2 x  1)  4 2x
 3x  6 and 4
 2nx 2  nx  4 3
∴  mx 2  (4  m  n) x  4  2nx 2  nx  4 x  2 and x6
By comparing the coefficients of x2 and x, we have ∴ The compound inequality has no solutions.
m  2n  (1)
 9. C
4  m  n  n  (2)
Let $x be the amount that Timothy pays for the TV set.
From (2), we have x(1  20%)(1  10%)  2700
4m  0
1.08x  2700
m  4
x  2500
By substituting m = 4 into (1), we have
∴ Timothy pays $2500 for the TV set.
(4)  2n
n2 10. B
Let b and h be the base and the height of the original
5. C triangle respectively.
Then, new base  b(1  x%) and new height  h(1  x%)
( x  k ) 2  k 2  2k  1
bh b(1  x%)  h(1  x%)
 (k  1) 2  (1  125%) 
2 2
x  k  k  1 or  (k  1)
2.25  (1  x%) 2
x  2k  1 or  1
1  x%  1.5
x  50

1 © Pearson Education Asia Limited 2016


2017 Mock Paper (Compulsory Part) - Paper 2 (Full Solutions)
11. C 14. B
( x  3z ) : ( y  2 z )  3: 2 Let T(n) be the number of dots in the nth pattern.
x  3z 3 T (1)  4
 T ( 2)  4  (1)  2  7
y  2z 2
2( x  3z )  3( y  2 z ) T (3)  7  ( 2)  2  11
2x  3y T ( 4)  11  (3)  2  16
x 3 T (5)  16  (4)  2  22
 T (6)  22  (5)  2  29
y 2
x  3y T (7)  29  (6)  2  37
( x  3 y ) : (2 x  y )  ∴ The number of dots in the 7th pattern is 37.
2x  y
x
3 15. D

y ∵ △ABC  △DEC (given)
2x ∴ AC = DC (corr. sides,  △s)
1
y = 12 cm
3 EC = BC (corr. sides,  △s)
3 = 5 cm
 2 AE = AC  EC
3
2   1 = (12  5) cm
2 = 7 cm
9 In △CDE,

8
DE  CD 2  CE 2 (Pyth. theorem)
 9:8
 12  5 cm
2 2

12. D  13 cm
k x In △ACD,
z , where k is a non-zero constant.
y2 AD  AC 2  CD 2 (Pyth. theorem)
2
k x
z2  4  12 2  12 2 cm
y
 288 cm
y4z2
 k2 ∴ The perimeter of △ADE
x
 AD  AE  DE
y4z2
∴ must be a constant.  ( 288  7  13) cm
x
 37.0 cm (cor. to 3 sig. fig. )
13. A
Maximum absolute error of the length of the longer wire
16. D
1
=  10 cm
2
= 5 cm
Lower limit of the actual length of the longer wire
= (20  5) cm
= 15 cm
Maximum absolute error of the length of the shorter wire
1
=  1 cm
2 With the notations in the figure,
= 0.5 cm consider △ADE and △ABC.
Upper limit of the actual length of the shorter wire DAE  BAC (common angle)
= (4 + 0.5) cm ADE  ABC (corr. s, DE // BC)
= 4.5 cm AED  ACB (corr. s, DE // BC)
∴ The lower limit of the value of x ∴ △ADE ~ △ABC (AAA)
= 15  4.5 a  b AC
= 10.5 ∴  (corr. sides, ~△s)
a AE
b 2 EC  EC
1 
a 2 EC
b 1

a 2

2 © Pearson Education Asia Limited 2016


2017 Mock Paper (Compulsory Part) - Paper 2 (Full Solutions)

1 19. C
 DE  a
Area of △ ADE 2 ∵ AC passes through the centre of the circle and

Area of △ DEF 1  DE  b BE = DE.
2 ∴ AE  BD (line joining centre to mid-pt. of chord
Area of △ ADE  2  area of △ DEF  chord)
i.e. ∠AED = 90°
 2  3 cm 2
In △ADE,
 6 cm 2 ∠DAE = ∠CBE (∠s in the same segment)
Area of △ ABC  a  b 
2 = 22°
  ADE  AED  DAE  180 (∠ sum of △)
Area of △ ADE  a 
2 ADE  90  22  180
3
Area of △ ABC     area of △ ADE ADE  68
2
9
  6 cm 2 20. C
4 Join AD.
 13.5 cm 2

17. D
In △ABD,
BD
sin  
AB  
BD  k sin 
ABD  ADB  BAD  180 (∠ sum of △)


3 AB  2 BC

ABD  90    180 ∴ 


AB

2

DBC  ABC  ABD


ABD  90   BC
 3

 90  (90   )

ACB AB
BAC
 BC
(arcs prop. to s at ☉ce)

In △BCD, 2
ACB  BAC
CD 3
tan DBC  In △ABC,
BD
CD  BD tan BAC  ACB  ABC  180 (∠ sum of △)
2
 sin   BAC  BAC  100  180
 k sin    3
 cos  
5
k sin 2  BAC  80
 3
cos  BAC  48
2
ACB  (48)  32
18. A
[sin(180   )  cos( )][sin(90   )  cos(270   )]  3

 ( sin   cos  )(cos   sin  ) ∵



2 BC  6 CD

 (sin   cos  ) 2
∴ 
CD

1
 (sin   2 sin  cos   cos  )
 3
2 2
BC
 (1  2 sin  cos  )
  1  2 sin  cos 
CAD CD
BAC
 
BC
(arcs prop. to s at ☉ce)

CAD 1

48 3
CAD  16
BCD  BAD  180 (opp. ∠s, cyclic quad.)
(32  ACD )  (48  16)  180
ACD  84

3 © Pearson Education Asia Limited 2016


2017 Mock Paper (Compulsory Part) - Paper 2 (Full Solutions)
21. D 25. D
For I:
1
∵ x-intercept of L1   0
a
∴ a > 0 ……(1)
1
∵ x-intercept of L2  0
c
∴ c > 0 ……(2)
With the notations in the figure, From (1) and (2), we have ac > 0.
∠YAB = 34° and ∠CAZ = 72° ∴ I is not true.
YAB  BAC  CAZ  180 (adj. ∠s on st. line) For II:
34  BAC  72  180 1
∵ y-intercept of L1    0
BAC  74 b
∵ AB = AC ∴ b < 0 ……(3)
∴ ∠ACB = ∠ABC (base ∠s, isos. △) 1
∵ y-intercept of L2   0
In △ABC, d
ABC  ACB  BAC  180 (∠ sum of △) ∴ d < 0 ……(4)
2ABC  74  180 From (3) and (4), we have bd > 0.
∴ II is true.
ABC  53
For III:
∠XBA = ∠YAB (alt. ∠s, BX // YA) ∵ Slope of L1 < slope of L2
= 34°
a c
∠XBC = ∠ABC  ∠XBA ∴  
= 53°  34° b d
= 19° a c

∴ The bearing of town C from town B is S19°W. b d
ad
c
22. A b
For I: ad  bc
(n  2)  180  162  n ∴ III is true.
180  n  360  162  n ∴ The answer is D.
18  n  360
n  20 26. A
∴ I is true. The equation of the circle C is:
For II:  x 2  y 2  4 x  ky  20  0
Size of each exterior angle x 2  y 2  4 x  ky  20  0
= 180°  162° (adj. ∠s on st. line)
 (4) (k )   k 
= 18° Centre of C    ,    2, 
∴ II is true.  2 2   2
For III: ∵ The centre of C lies in quadrant IV.
Number of diagonals ∴ k<0
17  20 2 2
  4  k 
2      (20)  5
 2   2 
 170
2
∴ III is not true. k
∴ The answer is A. 4     20  5
2
2
23. B k
The rectangular coordinates of the point P 24     5
2
 (2 cos120, 2 sin120) 2
k
 (1, 3 )   1
2
∴ The rectangular coordinates of its image
k
 (1,  3 )  1 or 1
2
k   2 or 2 (rejected)
24. D
1
∵ Slope of L1   and slope of L2 = 2
2
∴ L1 is not parallel to L2.
∴ The locus of P is a pair of perpendicular lines.

4 © Pearson Education Asia Limited 2016


2017 Mock Paper (Compulsory Part) - Paper 2 (Full Solutions)
27. B For III:
P(the product of two numbers on the dice is even) Case 1: x < 2
27 Arrange the 10 integers in ascending order:
 x 2 2 5 5 5 6 6 7 7
36
∴ Median = 5
3
 Case 2: x > 7
4 Arrange the 10 integers in ascending order:
Expected gain of the game 2 2 5 5 5 6 6 7 7 x
3  56
 $  12  4  ∴ Median 
4  2
 $5  5.5
∴ The value of c may not be greater than 5.
28. A ∴ III may not be true.
Only when the unit digit is 5, the 4-digit number is ∴ The answer is A.
divisible by 5 but not divisible by 10.
∴ The required probability Section B
10
 31. A
100
1 1 1 1 1
   
10 x 1
2
x  1 ( x  1)( x  1) x 1
1  ( x  1)

29. B ( x  1)( x  1)
Original value of the angle of the sector representing grade A x
 360  125  105  40  2
x 1
 90 x
Number of customers giving grade A 
1 x 2
90
 20 
360 32. B
5 30
Slope  3
New value of the angle of the sector representing grade A 0  (1)
54 log 2 y  3 log 2 x  3
 360  ∴
20  4
log 2 y  3 log 2 x  3
 135
∴ Increase in the angle of sector representing grade A y
log 2 3  3
 135  90 x
y
 45 8
x3
30. A y  8x3
∵ The range of the 10 integers is greater than 5. 33. A
∴ The value of x must be the least or the greatest among
10000110000 2  1 210  1 2 5  1 2 4
the 10 integers.
Case 1: The value of x is the least.  210  (2  1)  2 4
7x5  210  3  2 4
x2
Case 2: The value of x is the greatest. 34. D
x25     k
x7   2k
For I:  2   2  (   ) 2  2
22555 x6677
a  (  k ) 2  2( 2k )
10
45  x  k 2  4k

10 35. C
∵ In both of the above cases, the value of x cannot be 5. 1
∴ a5 z
ai
∴ I is true.
1 ai
For II:  
ai ai
∵ x cannot be 2, 6 or 7.
ai
∴ ‘5’ has the highest frequency among the  2
10 integers. a 1
∴ b=5 a 1
 2  2 i
∴ II is true. a 1 a 1
5 © Pearson Education Asia Limited 2016
2017 Mock Paper (Compulsory Part) - Paper 2 (Full Solutions)

a 2 ∵ T(2)  T(1)  T(3)  T(2)



a2 1 5 ∴ It is not an arithmetic sequence.
2a 2  5a  2  0 ∴ III is not true.
∴ The answer is A.
(2a  1)(a  2)  0
1 38. B
a or 2 (rejected)
2 x  y  1
1 
When a = , The system of inequalities are  x  0 .
2 y  0
1 
the imaginary part of z  2 ∵ x  y  1 , i.e. y  x  1 .
1
  1 ∴ The lower half-plane of y  x  1 , i.e. x  y  1 is
2
the solution of the inequality x  y  1 .
4
 x  y  1
5 
∴ Region III represents the solutions of  x  0 ,
y  0
36. B 
The graph of y = f (x) can be obtained by reflecting the a  b  1
graph of y = f (x) about the y-axis. 
i.e. a  0 .
The graph of y = f (x) + 1 can be obtained by translating
b  0
the graph of y = f (x) upwards by 1 unit. 
∴ Only option B may represent the two graphs. i.e. P(a, b) may lie in region III.

37. A 39. C
For I: sin   3 sin 3 
T (2)  T (1)  2  3m  (1  2m)
3 sin 3   sin   0
 m  1
sin  (3 sin 2   1)  0
T (3)  T (2)  3  4m  (2  3m)
 m  1 sin   0 or 3 sin 2   1  0
T (4)  T (3)  4  5m  (3  4m) 1 1
sin   0 or sin   or sin   
 m  1 3 3
∵ T(2)  T(1) = T(3)  T(2) = T(4)  T(3) When sin   0 ,
∴ It is an arithmetic sequence.   0 (rejected) or 180 or 360 (rejected)
∴ I is true. 1
For II: When sin   ,
3
T (2)  T (1)  log m 2  log m
  35.3 or 180  35.3
 m2 
 log   35.3 or 144.7

 m  1
 log m When sin    ,
3
T (3)  T (2)  log m  log m
3 2
  180  35.3 or 360  35.3
 m3   215.3 or 324.7
 log 2 

m  ∴ The equation sin  3sin 3  has 5 roots.
 log m
40. C
T (4)  T (3)  log m 4  log m 3
 m4 
 log 3 
m 
 log m
∵ T(2)  T(1) = T(3)  T(2) = T(4)  T(3)
∴ It is an arithmetic sequence.
∴ II is true.
For III:
With the notations in the figure,
T (2)  T (1)  2 m 2  2 m1 let r be the radius of the circle.
 2 m1 (2  1) Then, the coordinates of T are (r, r).
TM = TN = r.
 2 m1
When y = 0,
T (3)  T (2)  2 m 3  2 m 2 4 x  3(0)  12  0
 2 m  2 (2  1) x3
 2 m2 ∴ The coordinates of A are (3, 0).

6 © Pearson Education Asia Limited 2016


2017 Mock Paper (Compulsory Part) - Paper 2 (Full Solutions)
When x = 0, 43. B
4(0)  3 y  12  0 Number of different queues formed
y4  P28  8!
∴ The coordinates of B are (0, 4).  2 257 920
AM  3  r
AP  AM (tangent property) 44. C
 3 r The required probability
BN  4  r 3 7 2 6
   
BP  BN (tangent property) 5 8 5 8
 4r 33

In △OAB, 40
AB  OA2  OB 2 (Pyth. theorem)
45. D
(3  r )  (4  r )  32  4 2 For I:
7  2r  5 x1  x2    x50
∵  a1
 2r  2 50
r 1 x1  x2    x50  50a1
∴ AP : BP = (3  1) : (4  1) = 2 : 3 ∴ Sum of the remaining 48 numbers  50a1  x1  x25
 3(3) 2(4)   9 8  50a1  x1  a1
∴ Coordinates of P   ,  , 
 23 23 5 5  49a1  x1
49a1  x1
41. A a 2  a1   a1
48
Let M(a, b) be the orthocentre of △ABC. a x
∵ AB is a horizontal line and CM  AB .  1 1
48
∴ CM is a vertical line. x25  x1
∴ x-coordinate of M = x-coordinate of C 
48
i.e. a = 42
0
∵ AM  BC
∴ a2  a1
∴ Slope of AM  slope of BC = 1
∴ I is true.
32  b 32  8
  1 For II:
60  42 24  42 x x x x
32  b 3 ∵ b1  25 26 and b2  26 27
 2 2
18 4 x26  x27 x25  x26
27 ∴ b2  b1  
32  b  2 2
2 x27  x25
37 
b 2
2 0
37 ∴ b2  b1
∴ The y-coordinate of the orthocentre of △ABC is .
2
∴ II is true.
42. C For III:
∵ c1  x50  x1 and c2  x50  x2
∴ c2  c1  x50  x2  ( x50  x1 )
 x1  x2
0
∴ c2  c1
∴ III is true.
∴ The answer is D.
With the notations in the figure,
1
 335 cm 2  AE  2010 cm 3
3
AE  18 cm
AE
sin ABE 
AB
18

26
ABE  43.8 (cor. to the nearest 0.1)
∴ The angle between AB and the plane BCD is 43.8°.
7 © Pearson Education Asia Limited 2016

You might also like